Difference between revisions of "1967 AHSME Problems/Problem 12"

(Created page with "== Problem == If the (convex) area bounded by the x-axis and the lines <math>y=mx+4</math>, <math>x=1</math>, and <math>x=4</math> is <math>7</math>, then <math>m</math> equals: ...")
 
(Solution)
Line 8: Line 8:
 
\textbf{(E)}\ \text{none of these}</math>
 
\textbf{(E)}\ \text{none of these}</math>
  
== Solution ==
+
== Answer ==
 
<math>\fbox{B}</math>
 
<math>\fbox{B}</math>
  

Revision as of 22:59, 27 March 2023

Problem

If the (convex) area bounded by the x-axis and the lines $y=mx+4$, $x=1$, and $x=4$ is $7$, then $m$ equals:

$\textbf{(A)}\ -\frac{1}{2}\qquad \textbf{(B)}\ -\frac{2}{3}\qquad \textbf{(C)}\ -\frac{3}{2} \qquad \textbf{(D)}\ -2 \qquad \textbf{(E)}\ \text{none of these}$

Answer

$\fbox{B}$

See also

1967 AHSME (ProblemsAnswer KeyResources)
Preceded by
Problem 11
Followed by
Problem 13
1 2 3 4 5 6 7 8 9 10 11 12 13 14 15 16 17 18 19 20 21 22 23 24 25 26 27 28 29 30
All AHSME Problems and Solutions

The problems on this page are copyrighted by the Mathematical Association of America's American Mathematics Competitions. AMC logo.png